the circle below has center O, and its radius is 5 ft. Given that m AOB = 70, Find the area of the shaded region and the length of the arc ADB

The Circle Below Has Center O, And Its Radius Is 5 Ft. Given That M AOB = 70, Find The Area Of The Shaded

Answers

Answer 1

The area of the shaded region of the circle is 63.24 ft²

How to find the shaded area?

We know that the area of a circle of radius R is given by the formula:

Area = pi*R²

Where pi = 3.14

If we have only a section of a circle defined by an angle θ, the area of that section will be:

area = (θ/360°)*3.14*R²

Here we know that the radius is 5 feet, and that AOB (the non-shaded part) is 70°, then we will have:

θ = 360° - 70° = 290°

Then the area of the shaded region is:

area = (290°/360°)*3.14*(5 ft)² = 63.24 ft²

Learn more about area at:

https://brainly.com/question/24487155

#SPJ1


Related Questions

help please?!This sphere has a radius of 6 cm.

What is the surface area of the sphere?

Enter your answer, in exact form, in the box.

Answers

Answer:

Step-by-step explanation:

  It's[tex]288\pi in2[/tex]

Answer:

Step-by-step explanation:

Here is real answer :>

sixty percent of 800 students in a business statistics class are female. if you want to make probability estimates of the sample proportion of female students without applying the finite population correction factor, what minimal sample size of the random sample should be used?

Answers

Sixty percent of 800 students in a business statistics class are female. if you want to make probability estimates of the sample proportion of female students without applying the finite population correction factor, 368 is the minimal sample size of the random sample should be used

The appropriate sample size for probability estimates of the sample proportion of female students in the business statistics class is determined by the margin of error (m) and the level of confidence (Z).

The formula for sample size calculation is as follows:

N = [tex](Z^2\times p \times q) / m^2[/tex]

where

N is the sample size

Z is the z-score that corresponds to the level of confidence

p is the estimated proportion of female students

q is 1 - p (proportion of male students)m is the margin of error.

Since we don't have any margin of error, we can assume it to be a standard value of 5%. And a z-score of 1.96 is appropriate for a 95% level of confidence.

As a result, the sample size for estimating the sample proportion of female students in the business statistics class is given by

N = [tex](1.96^2\times0.60\times0.40) / (0.05^2)[/tex]

N = 368 students

For similar question on sample size

https://brainly.com/question/28583871

#SPJ11

Write the ratios for sin A and cos A. The diagram is not drawn to scale.
sin A= 14/50 cos A 48/50
sin A= 48/50 cos A 14/50
sin A 48/14 cos A 14/50
sin A= 48/50 cos A 14/48

Answers

The ratio of SINA and COSA = 48/50 and 14/50

What is trignometric ratios?

This is the boundary or contour length of a 2D geometric shape.

Depending on their size, multiple shapes may have the same circumference. For example, imagine a triangle made up of wires of length L.

The same wire can be used to create a square if all sides are the same length.

The length covered by the perimeter of the shape is called the perimeter. Therefore, the units of circumference are the same as the units of length.

As we can say, the surroundings are one-dimensional. As a result, you can measure in meters, kilometers, millimeters, etc.

Inches, feet, yards, and miles are other globally recognized units of circumference measurement.

According to our question,

sina = perpendicular\ hypotenuse

= 48/50

cosa= base\ perpendicular

=

14/50

Hence, The ratio of SINA and COSA = 48/50 and 14/50

learn more about trigonometric ratios click here:

brainly.com/question/1201366

#SPJ1

PLEASE HELPP I NEED HELP WITH THIS MATHS PLEASE

Answers

1. It will take about 8 years for the tithe to double in value if invested at 7.75% APR compounded monthly.

2. An annual interest rate of 4.84% compounded semi-annually would be required for $22,500 to accumulate to $50,000 in 14 years.

3 A principal of $26,512.18 invested in a GIC at 4% APR compounded quarterly would return $40,000 in 9 years.

How to solve the questions

a. Using the TVM Solver function in Excel, we can input the following information:

Present value (PV): -$35,000 (since it's an outgoing cash flow)

Future value (FV): $70,000 (since we want the tithe to double in value)

Interest rate per period (Rate): 7.75%/12 (since the APR is compounded monthly)

Number of periods (Nper): unknown (what we're solving for)

Payment (Pmt): 0 (since there are no recurring payments)

Solving for Nper, we get 96.16 months, or approximately 8 years.

Therefore, it will take about 8 years for the tithe to double in value if invested at 7.75% APR compounded monthly.

b. Present value (PV): -$22,500 (since it's an outgoing cash flow)

Future value (FV): $50,000

Interest rate per period (Rate): unknown (what we're solving for)

Number of periods (Nper): 14*2=28 (since the interest is compounded semi-annually, we need to double the number of years)

Payment (Pmt): 0

Solving for Rate, we get 4.84% APR.

Therefore, an annual interest rate of 4.84% compounded semi-annually would be required for $22,500 to accumulate to $50,000 in 14 years.

c. Using the TVM Solver function in Excel, we can input the following information:

Present value (PV): unknown (what we're solving for)

Future value (FV): $40,000

Interest rate per period (Rate): 4%/4 (since the APR is compounded quarterly)

Number of periods (Nper): 9*4=36 (since the interest is compounded quarterly, we need to multiply the number of years by 4)

Payment (Pmt): 0

Solving for PV, we get $26,512.18.

Therefore, a principal of $26,512.18 invested in a GIC at 4% APR compounded quarterly would return $40,000 in 9 years.

Learn more about APR on

https://brainly.com/question/2772156

#SPJ1

julio has $31.00 he earns half of that much mowing a lawn. How much money does he have in all?

Answers

Answer: $ 46.50

First divided 31 by 2

Which equals...

15.50

Then add 15.50 to 31.

46.50

The answer is $46.50

SOMEONE HELP PLEASE
Kiki runs 4 3/7 miles during the first week of track practice she runs 6 2/3 miles during the second week of track practice. How much longer does kiki run during the second week of track practice than the first week of track practice

Answers

Total longer distance is [tex]3\frac{2}{21} miles[/tex], that kiki run during the second week of track practice than the first week of track practice.

We have, a runner Kiki and she runs on track for practice. In first week,

Distance runs by Kiki on track practic = [tex]4 \frac{3}{7} miles[/tex]

which is a mixed fraction so, simplify it into simple fraction that is 25/7 miles. In second week,

Distance runs by Kiki on track practice = [tex]6 \frac{2}{3} miles[/tex]

After simplification, it is equals to 20/3 miles. We have to calculate the longer distance that kiki run during the second week of track practice than the first week of track practice. Let the distance run during second week be longer by 'x miles' from distance run during first week. The required distance can be calculated by difference between the distance that kiki run during the second week of track practice and the first week of track practice. So, [tex]x = \frac{20}{3} - \frac{25}{7 }[/tex].

taking least common multiple of (3,7)= 21

=> [tex] x = \frac{20× 7 - 3× 25}{21} [/tex]

[tex] =>x = \frac{ 140 - 75}{21} = \frac{65}{21 }[/tex].

Hence, required distance value is

[tex]3\frac{2}{21} [/tex].

For more information about distance, visit :

https://brainly.com/question/28551043

#SPJ4

12.8 x 3/4 also One more is
One-fifth the sum of one-half and one-third this one u have to write in equivalent expression

Answers

Answer:

\

Step-by-step explanation:

Given expression: 3x+9

Take 3 outside from the expression, we get,

= 3(x+3), which is called the equivalent expresion

The graphs of line a and b are shown in this coordinate grid
Match each line with it's equation. Drag each equation to the corresponding box for each line

Answers

The correct option for the given graph is option 1 and option 3. The equation matches the intersection point (1,1).

For option 1: intersection point (1,1)

substitute the values of x & y in the given equation.

1 = 3 (1) - 2

1 = 1

LHS = RHS

For option 2: point (1,1)

substitute the values of x & y in the given equation.

1 = 2 (1) + 3

1 = 5

LHS ≠ RHS

For option 3: point (1,1)

substitute the values of x & y in the given equation.

1 = -2 (1) + 3

1 = 1

LHS = RHS

For option 4: point (1,1)

substitute the values of x & y in the given equation.

1 = - [tex]\frac{1}{2}[/tex] (1) + 3

1 = [tex]\frac{5}{2}[/tex]

LHS ≠ RHS

For option 5: point (1,1)

substitute the values of x & y in the given equation.

1 = - [tex]\frac{1}{3}[/tex] (1) + 3

1 = [tex]\frac{8}{9}[/tex]

LHS ≠ RHS

Therefore, correct option for the given graph is option 1 and option 3.

Learn more about Graphs:

https://brainly.com/question/29192564

#SPJ1

what's a drawback to using a histogram?

Answers

Answer:

Step-by-step explanation:

One potential drawback of using a histogram is that it can be sensitive to the choice of bin width or bin size. If the bin size is too small, the histogram may appear too noisy or have too many empty bins, which can obscure patterns in the data. If the bin size is too large, important features of the distribution may be lost or smoothed out. Additionally, histograms do not always show the actual values of the data points, but rather a summary of the data. This means that some details about the data may be lost, such as the exact values of outliers or individual data points.

function “p” is in the form y = ax 2+c. if the values of “a” and “c” are both less than 0. which graph could represent “p”?

Answers

Since the value of a is less than 0, the graph of the parabola would be opening downwards. Because of this we can rule out option C. In a quadratic equation, c represents the y-intercept, and, in this case c is negative, meaning the y-intercept is less than 0. Only option B has a downward-opening curve and a y-intercept less than 0, so it is the answer.

Answer: p= 2+c

Step-by-step explanation:

11. Hannah recorded the number of miles she jogged. She started jogging 3 41 miles. Every week she jogged an additional
1 21 miles. Select all true statements. The y-intercept is the rate of change. The y-intercept is the starting value. The slope is 121. The slope is 3 41. The y-intercept is 1 21. The y-intercept is 3 41

Answers

Using coordinate geometry,

The y-intercept is the starting value (true).The slope is 1/2 or 0.5, not 121 or 3/41 (false).The y-intercept is 3/41 is also false as it contradicts the first true statement, therefore, the y-intercept is the starting value and the y-intercept of 3/41 is also true.

The problem provides two pieces of information about Hannah's jogging routine: she started with 3 41 miles and added 1/21 miles every week. We can use this information to create a linear equation that represents the number of miles Hannah jogged as a function of the number of weeks she has been jogging.

To create this equation, we first identify the starting value, which is 3 41 miles. This is also the y-intercept of the line. Next, we determine the slope of the line, which is the rate at which the number of miles increases each week. The slope is equal to the change in y (miles) divided by the change in x (weeks), which in this case is (1/21 - 3/41)/1 = -2/1 = -2. Therefore, the slope of the line is -2.

Putting these two pieces of information together, we get the equation y = -2x + 3 41, where y is the number of miles jogging and x is the number of weeks of jogging. This is a linear equation in slope-intercept form, where the slope is -2 and the y-intercept is 3 41.

Learn more about the co-ordinate geometry at

https://brainly.com/question/21094061

#SPJ4

9. A helicopter spots two landing pads below. The straight-line distance
from the helicopter to the pads is 14 miles to Landing Pad A and 8 miles
to Landing Pad B. If the landing pads are 20 miles apart, find the angle
of depression from the helicopter to Landing Pad B.

Answers

In response to the stated question, we may state that As a result, the Pythagorean theorem angle of depression from the helicopter to Landing Pad B is around 16.7 degrees.

what is Pythagorean theorem?

The Pythagorean theorem is a fundamental mathematical principle that describes the connection between the sides of a right triangle. It asserts that the sum of the squares of the other two sides' lengths equals the square of the hypotenuse's length (the side opposite the right angle). The mathematical theorem is as follows: c2 = a2 + b2 Where "c" denotes the hypotenuse length and "a" and "b" indicate the lengths of the other two sides, known as the legs.  

We can calculate the angle of depression from the helicopter to Landing Pad B using trigonometry.

Let we make a diagram:

            B

              /|

             / |

            /  |  8 miles

        14 /   |

          /θ  |

         /    |

        /     |

       /______|______ A

             20 miles

Using trigonometry, we can deduce:

tan(θ) = opposite / adjacent

The opposing side in this example is 14 - 8 = 6 miles (since the helicopter is 14 miles from Landing Pad A and 8 miles from Landing Pad B, and the pads are 20 miles apart). The distance between the two sides is 20 miles (the distance between the landing pads).

tan(θ) = 6 / 20

tan(θ) = 0.3

θ = arctan(0.3)

θ ≈ 16.7 degrees

As a result, the angle of depression from the helicopter to Landing Pad B is around 16.7 degrees.

To know more about Pythagorean theorem visit:

https://brainly.com/question/14930619

#SPJ1

Determine the magnitude of the force P for which the resultant of the four forces acts on the rim of the plate. Given: F= 320 N. 30° 120 N 80 N P x 250 mm F 7 The magnitude of the force P is N.

Answers

The magnitude of the force P is 464.77 N.

STEP BY STEP EXPLANATION:


Step 1: Break down each force into components.
F = 320 N at 30°
Fx = F * cos(30°) = 320 * cos(30°) = 277.13 N (horizontal)
Fy = F * sin(30°) = 320 * sin(30°) = 160 N (vertical)

120 N is in the horizontal direction (assume positive x-direction):
Fx2 = 120 N

80 N is in the vertical direction (assume positive y-direction):
Fy2 = 80 N

Step 2: Sum up the components.
Total horizontal force (Fxtotal) = Fx + Fx2

= 277.13 + 120 = 397.13 N


Total vertical force (Fytotal) = Fy + Fy2

= 160 + 80 = 240 N

Step 3: Find the magnitude of the resultant force.
            Resultant force (R) = sqrt(Fxtotal^2 + Fytotal^2)

= sqrt(397.13^2 + 240^2) = 464.77 N

Step 4: Determine the magnitude of the force P.
Since the resultant of the four forces should act on the rim of the plate, it means that the force P should be equal in magnitude and opposite in direction to the resultant force R.

The magnitude of the force P is 464.77 N.

To know more about Magnitude:

https://brainly.com/question/30033702

#SPJ11

f scores are normally distributed with a mean of 35 and a standard deviation of 10, what percent of the scores is: (a) greater than 34?

Answers

The percentage of scores greater than 34 is 0.5398 or 53.98%.

Given that the mean of scores (μ) = 35 and the standard deviation (σ) = 10. We need to find the percentage of scores greater than 34. Since the scores are normally distributed, we can standardize the variable by using the z-score formula.

z = (x - μ) / σ

Here, x = 34, μ = 35 and σ = 10z = (34 - 35) / 10z = -0.1

We need to find the area to the right of the z-score line on the standard normal distribution table. The standard normal distribution table provides the probabilities corresponding to the z-scores, i.e. the area under the curve to the right or left of the z-score line on the distribution table. The area to the right of the z-score line represents the percentage of scores that are greater than the given value. Using the standard normal distribution table, the area to the right of the z-score line -0.1 is 0.5398.

The percentage of scores greater than 34 is 0.5398 or 53.98%.

To learn more about percentage of scores refer :

https://brainly.com/question/25361715

#SPJ11

a 336-m long fence is to be cut into pieces to make three enclosures, each of which is square. how should the fence be cut up in order to minimize the total area enclosed by the fence?

Answers

The fence ought to be cut into 12 pieces, every one of length 28 m, to make three squares, each with a side length of 28 m. This will limit the total area encased by the fence.

To limit the total area encased by the fence, the three squares ought to have equivalent areas. Let x be the length of each side of the squares. Then the perimeter of each square is 4x, and the total length of the fence is 3(4x) = 12x. Since the total length of the fence is given to be 336 m, we have:

12x = 336

Addressing for x, we get:

x = 28

Find out more about fence

brainly.com/question/14785438

#SPJ4

there are 3 soccer games in a month, and 8 are played at night. the season is 4 months. how many games are the season?

Answers

There are a total of 48 soccer games in the season.

Since there are 3 soccer games in a month, there will be 12 games in a season (3 games/month x 4 months). Since 8 games are played at night and assuming that all games are played either during the day or at night, we can calculate the number of games played during the day as:

Number of day games = Total number of games - Number of night games

= 12 games/month x 4 months - 8 night games/month x 4 months

= 48 games - 32 games

= 16 games

Therefore, the total number of games in the season is:

Total number of games = Number of day games + Number of night games

= 16 games + 32 games

= 48 games

So, there are 48 soccer games in the season.


To know more about soccer, refer here:

https://brainly.com/question/15499943#

#SPJ11

Triangle ABC has vertices at A(−4, 3), B(0, 5), and C(−2, 0). Determine the coordinates of the vertices for the image if the preimage is translated 4 units down.

A′(−4, −1), B′(0, 1), C′(−2, −4)
A′(−4, 7), B′(0, 9), C′(−2, 4)
A′(0, 3), B′(4, 4), C′(3, 0)
A′(−8, 7), B′(−4, 9), C′(−6, 4)

Answers

The coordinates of the vertices for the image if the preimage is translated 4 units down are A′(-4, -1), B′(0, 1), C′(-2, -4).

What is meant by preimage?

In geometry, a preimage is the original figure or shape before any transformation is applied. It is the initial configuration of the object that is being transformed. For example, if we have a square and we rotate it by 90 degrees, the original square is the preimage and the resulting figure after the rotation is the image.

To translate the preimage 4 units down, we need to subtract 4 from the y-coordinates of all vertices. Therefore, the coordinates of the image vertices are:

A′(-4, 3-4) = (-4, -1)

B′(0, 5-4) = (0, 1)

C′(-2, 0-4) = (-2, -4)

Therefore, the vertices of the image triangle are A′(-4, -1), B′(0, 1), and C′(-2, -4).

So, the correct option is: A′(-4, -1), B′(0, 1), C′(-2, -4).

To learn more about preimage visit:

https://brainly.com/question/30093252

#SPJ1

For any acute angle A if sin A = 2x-1/2x+1, what is the value of cos A cot A?

Answers

The trigonometric expression cosAcotA = 8x/(4x² - 1)

What is a trigonometric expression?

A trigonometric expression is an expression that contains trigonometric ratios.

Given that for any acute angle A if sin A = 2x-1/2x+1, we desire to find the value of cos A cot A?

So, we proceed as follows

cosAcotA = cosA × cosA/sinA  (since cotA = cosA/sinA)

= cos²A/sinA

Now using the trigonometric identity

sin²A + cos²A = 1

⇒ cos²A = 1 - sin²A

So, substituting this into the equation, we have that

cosAcotA = cos²A/sinA

=  (1 - sin²A)/sinA

= 1/sinA - sin²A/sinA

= 1/sinA - sinA

Substituting the value of sinA into the equation, we have

= 1/(2x - 1)/(2x + 1) - (2x - 1)/(2x + 1)

= (2x + 1)/(2x - 1) - (2x - 1)/(2x + 1)

Taking the L.C.M, (2x - 1)(2x + 1), we have

= [(2x + 1)² - (2x - 1)²]/[(2x - 1)(2x+ 1)]

=  [(2x + 1)² - (2x - 1)²]/[(2x)² - 1²)]

=  [(2x + 1 + 2x - 1)(2x + 1 - (2x - 1)]/(2x)² - 1²)

=  [(2x + 1 + 2x - 1)(2x + 1 - 2x + 1)]/4x² - 1)

=  [(4x)(2)]/4x² - 1)

= 8x/(4x² - 1)

So, cosAcotA = 8x/(4x² - 1)

Learn more about trigonometric expresion here:

https://brainly.com/question/29321762

#SPJ1

ten chairs are arranged in a circle. find the number of subsets of this set of chairs that contain at least three adjacent chairs.

Answers

Ten chairs are arranged in a circle. The number of subsets of this set of chairs that contain at least three adjacent chairs is 310.

The given that 10 chairs arranged in a circle.

            Now we have to find the number of subsets of this set of chairs that contain at least three adjacent chairs.

           To solve this, we can use the concept of permutations and combinations. The first step is to consider the number of ways in which three chairs can be selected and arranged in a subset that is adjacent to each other.

           This can be done in 10 different ways, as there are 10 chairs in total and we can select any one of them as the starting point.

           The next step is to consider the number of ways in which we can add additional chairs to this subset. For example, we can add a fourth chair to the subset in two different ways: either to the left of the first chair or to the right of the third chair.

            Similarly, we can add a fifth chair to the subset in four different ways, a sixth chair in six different ways, and so on. Using this logic, we can create the following table:

                    Length of subset number of ways to select the subset number of ways to add chairs

           Total number of subsets31 (adjacent)

                                = 10  ---43 (adjacent) 10*2

                                =20---55 (adjacent)10*4

                                =40---67 (adjacent)10*6

                                =60---79 (adjacent)10*8

                               =80---810 (adjacent)10*10

                               =100---

              As we can see from the table, the total number of subsets that contain at least three adjacent chairs is given by:

           Total number of subsets = 10 + 20 + 40 + 60 + 80 + 100

                                                     = 310

       Therefore, the number of subsets of this set of chairs that contain at least three adjacent chairs is 310.

Learn more about the number of subsets of chairs at: https://brainly.com/question/17514113

#SPJ11

ANSWER THIS WITHIN ONE HOUR OR ELSE I WILL BE DOOMED
mARKING BRAINLIEST
PS: FILL IN THE SHEET DONT JUST GIVE AN ANSWER

Answers

The prompt on fractions is given as follows:

1 ) (1/2) ÷ 4 = 1/8
2) ( 1/5) ÷ 2 = 1/10

3) (1/3) ÷ 5 = 1/15

4) (1/4) ÷ 4 = 1/16
5) The solution to the puzzle for (1/2) ÷ 3 is given below.

What is the calculation for the above equations?

The calculations are given as follows;

1 )

= (1/2) x (1/4) [dividing by a number is the same as multiplying by its reciprocal]

= 1/8

2) (1/5) ÷ 2

= (1/5) x (1/2)

= 1/10

3)

(1/3) ÷ 5

= (1/3) x (1/5)

= 1/15

4) (1/4) ÷ 4

= (1/4) x (1/4)

= 1/16

5) Mary had 1/2 of a pie that she wanted to share with 3 of her friends. She decided to divide it equally among them. Each friend got 1/6 of the pie. To check, Mary multiplied 1/6 by 3 and got 1/2. This shows that (1/2)/3 equals 1/6, since dividing by 3 is the same as multiplying by its reciprocal, 1/3.


Learn more about Fractions on:
https://brainly.com/question/10708469
#SPJ1

for the beam and loading shown, (a) draw the shear and bending-moment diagrams, (b) determine the equations of the shear and bending-moment curves. 5.1

Answers

Bending moment curve equation below point A will be:

M = 15x - 3x² for 0 ≤ x ≤ b

Determination of shear and bending moment curves.

For the beam and loading shown, we can do the following:

Equation of shear curve (above point A):V = RA - w.x

For x = a,V = RA - w.a

For x = b,V = RA - w.b

Since the loading is symmetric, RA = w(a + b) / 2= (6 * 5) / 2= 15kNV = 15 - 6a for a ≤ x ≤ b

Equation of shear curve (below point A):

V = RA - w.x

For x = 0,V = RA - w.0RA = w(a + b) / 2= (6 * 5) / 2= 15kNV = 15k for 0 ≤ x ≤ a

The shear curve equation becomes;

V = 15k for 0 ≤ x ≤ a

V = 15 - 6a for a ≤ x ≤ b

Equation of bending moment curve (above point A):

M = RAx - ½w.x²For 0 ≤ x ≤ a,

M = 15x - ½(6x²) = 15x - 3x²For a ≤ x ≤ b,

M = 15x - 6a(x - a) - ½(6x²)= 15x - 6ax + 6a² - 3x²

The bending moment curve equation above point A becomes:

M = 15x - 3x² for 0 ≤ x ≤ a

M = 15x - 6ax + 6a² - 3x² for a ≤ x ≤ b

Equation of bending moment curve (below point A):

M = RAx - ½w.x²For 0 ≤ x ≤ b,

M = 15x - ½(6x²) = 15x - 3x²

The bending moment curve equation below point A becomes;

M = 15x - 3x² for 0 ≤ x ≤ b

Learn more about Bending Moment.

brainly.com/question/30242055

#SPJ11

Please help 30 points I've been struggling

Identify the Slope and y - intercept from the graph

Slope (m) =
b =

Write the equation of the line in Slope-Intercept form

Answers

The slope is 2, and the y-intercept is 4.

Select the correct answer.

Answers

The subtraction property of equality is used for the justification of step 2 in the solution.

How to use the subtraction property of equality?

The subtraction property of equality states that subtracting the same number from both sides of an equation does not affect the equality.

Therefore, Justify the property used for step 2 in the equality.

1 / 2 r + 1 / 2  = - 2 / 7 r + 6 / 7  - 5

Step 1 : 1 / 2 r + 1 / 2  = - 2 / 7 r - 29 / 7

Step 2: 1 / 2 r = - 2 / 7 r - 65 / 14

We had to subtract 1 / 2 from both sides of the equation to arrive at step 2. Therefore, the subtraction property of equality is the justification for step 2 in the solution.

learn more on subtraction property here:https://brainly.com/question/9070018

#SPJ1

I need help with this problem. Joe bought a gallon of gasoline for 2. 85 per gallon and c cans of oil for 3. 15 per can

Answers

From the given information provided, the expression that need to determine the total amount is Total cost = $2.85/gallon x g gallons + $3.15/can x c cans.

The expression that can be used to determine the total amount Joe spent on gasoline and oil is:

Total cost = Cost of gasoline + Cost of oil

We can represent the cost of gasoline as:

Cost of gasoline = price per gallon x number of gallons

Substituting the given values, we get:

Cost of gasoline = $2.85/gallon x g gallons

Similarly, we can represent the cost of oil as:

Cost of oil = price per can x number of cans

Substituting the given values, we get:

Cost of oil = $3.15/can x c cans

Putting it all together, we get:

Total cost = $2.85/gallon x g gallons + $3.15/can x c cans

Expression that can be used to determine the total amount Joe spent on gasoline and oil is:

Total cost = $2.85/gallon x g gallons + $3.15/can x c cans

Question - Joe bought g  gallons of gasoline for $2.85 per gallon and c  cans of oil for  $3.15 per can. What expression can be used to determine the total amount Joe spent on gasoline and oil?

Learn more about amount here: brainly.com/question/25720319

#SPJ4

The graph below shows a company's profit f(x), in dollars, depending on the price of goods x, in dollars, being sold by the company:
f(x)
150
120

Part A: What do the x-intercepts and maximum value of the graph represent in context of the described situation?

Part B: What are the intervals where the function is increasing and decreasing, and what do they represent about the sale and profit for the company in the situation
described?

Part C: What is an approximate average rate of change of the graph from x= 1 to x= 3, and what does this rate represent in context of the described situation?

Answers

The vertical axis of the graph represents profit, so the x-intercepts represent prices in the produce 0 profit. The maximum value of the graph is the maximum profit that can be obtained for anyprice

How to explain the graph

The higher or largest number of the chart is the maximum reach

B) We read the value of f(1) from the graph to be about 120, so the average rate of change is about:

(f(4) -f(1))/(4 -1) = (270 -120)/(3) = 50

The average rate of the change from x = 1 to x = 4 is about 50.* This means profit will increase on average $50 for each $1 increase in price in what interval.

If we take the peak profit to be $270 we can write f(x) as:

f(x) 16.875x(x-8)

Then f(1) = 118.15 and average rate of change is 50.625.

Learn more about graph on;

https://brainly.com/question/25184007

#SPJ1

a die is rolled twice and the sum of numbers appearing on the upper faces of them is observed to be 7. what is the probability that the number 2 has appeared atleast once? hint: use the concept of conditional probability)

Answers

The probability of getting at least one 2 given that the sum of the numbers is 7 is 2/6 or 1/3.

To find the probability that the number 2 has appeared at least once given that the sum of the numbers is 7, we need to use the concept of conditional probability.

Let's consider the possible outcomes when two dice are rolled. The total number of outcomes is 36, as each die has six possible outcomes.

Out of these 36 outcomes, there are six outcomes in which the sum of the numbers appearing on the upper faces is 7. These outcomes are (1,6), (2,5), (3,4), (4,3), (5,2), and (6,1).

Out of these six outcomes, there are two outcomes in which the number 2 appears at least once: (1,6) and (2,5).

We can use the formula for conditional probability to verify our answer:

P(2 appears at least once | sum is 7) = P(2 appears at least once and sum is 7) / P(sum is 7)

P(2 appears at least once and sum is 7) = 2/36 = 1/18 (as there are two outcomes with a sum of 7 that have a 2 in them)

P(sum is 7) = 6/36 = 1/6

So, P(2 appears at least once | sum is 7) = (1/18) / (1/6) = 1/3, which is consistent with our previous answer.

To learn more about probability click on,

https://brainly.com/question/13866543

#SPJ4

the area of the figure

Answers

Answer:

432

Step-by-step explanation:

go 16x27 and that is your answer

Answer:

The area of the figureo will be 432

BECAUSE :-

You have to multiply 16 × 27 = 432

Hope Its Help You !!

the product of two consecutive positive integers is 3 less than three times their sum find the integers

Answers

The two consecutive positive integers are 5 and 6.

Let the two consecutive positive integers be x and x + 1. We are given that the product of these integers is 3 less than three times their sum. This can be expressed as:
[tex]x(x + 1) = 3(x + x + 1) - 3[/tex]
Now we can solve for x:
[tex]x^2 + x = 6x + 3 - 3[/tex]
[tex]x^2 + x = 6x[/tex]
[tex]x^2 - 5x = 0[/tex]
Factoring the left side of the equation, we get:
[tex]x(x - 5) = 0[/tex]
From this equation, x can be 0 or 5.

However, since the question asks for positive integers, we can't use x = 0. Therefore, x = 5.

for such more questions on positive integers

https://brainly.com/question/30076540

#SPJ11

Subtract the following polynomials.

Answers

The subtraction of the polynomials (3.1x + 2.8z) - (4.3x - 1.2z) is -1.2x + 4x

How to subtract polynomials?

A polynomial is an expression consisting of a sum of a finite number of terms, each term being the product of a constant coefficient and one or more variables raised to a non-negative integer power.

(3.1x + 2.8z) - (4.3x - 1.2z)

open parenthesis

3.1x + 2.8z - 4.3x + 1.2z

combine like terms

3.1x - 4.3x + 2.8z + 1.2z

-1.2x + 4x

Ultimately, -1.2x + 4x is the results of the subtraction of the polynomial.

Read more on polynomials:

https://brainly.com/question/4142886

#SPJ1

Answer

D =

E =

Please help

Answers

Answer:

d = 3.75

e = 8/3

Step-by-step explanation:

8/6 = 5/d = e/2

4/3 = 5/d

4d = 3 × 5

d = 3.75

4/3 = e/2

3e = 4 × 2

e = 8/3

Other Questions
Tim Anjaan took up the job of cleaning the windows in a row house the first floor had 8 Windows in a row and ground floor at 7 Tim behind lazy of open the cleaning clean the ground floor Windows and town will get to up the job of cleaning the first floor Windows the master was very happy and paid them 300 rupees Tim was happy that he would get 150 rupees and also plan how to spend it but he thought that I eat would be unfair to share the same amount when Tom had clean more number of Indus solution Tom gave him four steps to find the correct share of both of them depending up on the work done step one find the money other stock has a beta of 1.10, the expected return on the market is 12 percent, and the risk-free rate is 3.6 percent. what must the expected return on this stock be? Please help, I got this and I dont know it Select the correct answer. Coles age is 3 years less than his sister Tinas age, t. If Cole is 18, which equation represents this situation, and how old is Tina? A. The equation that represents this situation is t 3 = 18. Tina is 21. B. The equation that represents this situation is t + 3 = 18. Tina is 15. C. The equation that represents this situation is 3 t = 18. Tina is 21. D. The equation that represents this situation is -3 t = 18. Tina is 15. when a company offers video-game consoles at a steep discount, but charges customers high fees for games, they are operating on a blank business model. multiple choice question. the cost of producing units of stuffed alligator toys is . find the marginal cost at the production level of units. as of 2020, the u.s. department of health and human services defined a poverty-level income for a family of four as how much annually? the elephant population in northwestern namibia and etosha national park can be predicted by the expression 2 , 649 ( 1.045 ) b , where b is the number of years since 1995. what does the value 2,649 represent? 3. DYSENTERY: ILLNESS a) insect: flyb) germs: bacteria c) waltz: danced) boat: yacht the d flip-flop can be build from two back-to-back s/r latches controlled by complementary clocks. question 20 options: true false what is the purpose of attraction content? 1 point to write content that attracts the right target reader who typically is not familiar with your company or brand to write content that is attractive to look at to write content that attracts as many readers as possible the end of the story for sarah's keys book holocaust (1 long minute answer!) which of the following is a defining feature of the medieval catholic church: group of answer choices religious force and political institution heresy the church was the largest land holder inquisition excommunication how does fitzgerald use the shifts in perspective (why include the reporter, why is nick seeing things through daisys eyes)? determine the capacitance of a teflon-filled parallel-plate capacitor having a plate area of 1.80 cm2 and a plate separation of 0.020 0 mm. what is the kinetic energy, in joules, of each ejected electron when light of 258.0 nm strikes the metal surface? i need help asap +10 points and brainliest. 16. aw computing would like to improve its case lightning record page by including: --a filtered component to display a message in bold font when a case is saved as a critical record type. ---a quick way to update the account status from the case layout. which two components should an administrator use to satisfy these requests? choose 2 answers Which of the following is not one of the key characteristics of good market research?ethicalaccuracytimelinessreliability if only one party to an insurance contract has made a legally enforceable promise, what kind of contract is it?